LSAT and Law School Admissions Forum

Get expert LSAT preparation and law school admissions advice from PowerScore Test Preparation.

 Administrator
PowerScore Staff
  • PowerScore Staff
  • Posts: 8916
  • Joined: Feb 02, 2011
|
#82054
Complete Question Explanation

The correct answer choice is (E).

Answer choice (A):

Answer choice (B):

Answer choice (C):

Answer choice (D):

Answer choice (E): This is the correct answer choice.


This explanation is still in progress. Please post any questions below!
 15veries
  • Posts: 113
  • Joined: Sep 25, 2016
|
#30512
Why is E wrong?
Since the author says "it may therefore be effective only when the negative information can be framed positively"
So I thought it requires experience to do so.
Is this unwarranted assumption like LR?
So something like "to frame positively, it requires experience..."?
 Adam Tyson
PowerScore Staff
  • PowerScore Staff
  • Posts: 5153
  • Joined: Apr 14, 2011
|
#30614
The answer to question 14 on that passage is E, 15, but I think perhaps you meant to ask about question 13, the one about the author's attitude? Assuming that's so, the problem with answer E is that it requires the very assumption that you made here - that framing it properly requires experience.

While that might be a reasonable precaution in the real world - not putting yourself in the hands of an inexperienced criminal defense attorney if your case will involve some complicated issues that require deft handling - it is not an absolute requirement. A fresh-faced first year associate might be a natural at something like this, maybe because she has a background in psychology or because she has been mentored by someone with a lot of experience. Maybe just because she has good instincts, too.

The point is that the passage never tells us explicitly that experience is mandatory, and we have to rely on the text of the passage and not on our assumptions about what that text might mean. This is a form of Must Be True question, so we have to pass the Fact Test, with our chosen answer based solely on the facts given and rejecting answers that rely on outside information. Since E does rely on outside info, we must reject it.

Besides, answer D is ever so much better! It tracks with the entire passage, the tone, the facts and details throughout. Even if we make the assumption required by E, answer D has so much more support in the passage that it fits our criteria for selecting the best answer, rather than one that is merely good.

Keep up the good work!
 edacyu
  • Posts: 11
  • Joined: Apr 17, 2018
|
#45939
Hi,

I selected in C in this question instead of E. While I now realize that E is correct, I’m looking to confirm that answer choice C was wrong because it was simply a partial match (stimulated situations is accounted for in the passage but not records of judges decisions in court cases).

Please let me know and thanks!
 Malila Robinson
PowerScore Staff
  • PowerScore Staff
  • Posts: 296
  • Joined: Feb 01, 2018
|
#46313
Hi edacyu,
Yes, as you stated Answer C is partially correct, there are no mentions of judges' decisions, but studies of simulated courtroom situations are mentioned in lines 19-20.
Hope that helps!
Malila
User avatar
 RichieC
  • Posts: 9
  • Joined: May 11, 2021
|
#89511
Hi,
I ended up choosing D, probably because I rushed, but after reviewing I can see why E is correct.

Still, I justified D in my head by saying "informal observations of nontrial uses of techniques analogous to stealing thunder" referred to the psychological experiments mentioned in Lines 28-33. Similarly, I justified "controlled studies of lawyers' courtroom behavior" with "simulated trial" mentioned in Lines 11-15 (13 to be specific).

Can someone tell me why my justifications were wrong?

Thanks in advance,
User avatar
 Beatrice Brown
PowerScore Staff
  • PowerScore Staff
  • Posts: 75
  • Joined: Jun 30, 2021
|
#89949
Hi Richie! Happy to help with this question :)

First, let's find the support for the correct answer choice, which is answer choice (E). In the first sentence of the second paragraph, the author discusses the results of "studies involving simulated trial situations," which matches the second half of answer choice (E) (which directly mentions "simulated trials"). A few sentences later, the author draws on results from psychological research, which matches the first half of answer choice (E), since psychological research would count as research not directly concerned with legal proceedings. Answer choice (E) is therefore strongly supported by the second paragraph of the passage.

Now, let's look at why answer choice (D) is incorrect. The beginning of the second paragraph says there has not been any empirical research directly testing the effectiveness of stealing thunder. Therefore, the second half of answer choice (D) ("controlled studies") is actually contradicted by the stimulus; "simulated trials" are not the same thing as "controlled studies" (a controlled study would have an intervention group and a control group, like a scientific research trial). For the first half of answer choice (D), the passage does not mention the use of techniques analogous to stealing thunder, as the psychological research discussed in the second paragraph doesn't necessarily count as the use of a "technique."

Although we can possibly stretch the information in the passage to fit answer choice (D), answer choice (E) is more clearly supported by the information in the passage. For Reading Comprehension, we always want to find the answer choice that is most clearly supported by the information in the passage without stretching the information in the passage to fit the answer choice.

I hope this helps, and let me know if you have any other questions!

Get the most out of your LSAT Prep Plus subscription.

Analyze and track your performance with our Testing and Analytics Package.